Question

Hi would you mind answering the question?

I appreciate your help.

(Matlab) - using matlab program

1) Write a MATLAB program to solve for the currents i 12, and i3 (amps) in the circuit shown below in which the resistances are R1 20, R2 13, R3 E 6, R4 32 and R5 -110 ohms and the input voltage Vi is 120 volts Derive the equations using the Kirchoff Voltage Law (sum of voltages around a loop must be zero). Format the output and display the currents with 3 decimal places Hint: The voltage on an edge in terms of the currents and a resistor are given below. There are three loops: the top box, and the lower left and right boxes. The sum of the voltages around the lower left loop is: The upper loop is: And the lower right loop is: R3 R4 (13-il) Self check: Loop current il is x.6xx amps Loop current i2 is x.x4x amps Loop current 13 is 0.x4x amps Extra credit: What happens if R2 and R4 are both set to zero? When checking around the loop of 12 12 When checking around the loop of i2 RA i3

0 0
Add a comment Improve this question Transcribed image text
Answer #1

Answer:

oms belo 213 the loap olu as bel 4511-1312-3277-120-> (D he loop ol t is as belou 2 (20)-z-3A, the toop o 16 -110-32i33 Meho in MATLA is au belouo 32 648 E İs Constant matrix-OF-rt20 3-39 3The Matlab code to solve the equations 1,2 and 3 is as below

MATLAB CODE:-

A=[45 -13 -32;13 -39 6; 32 6 -148];
B=[120;0;0];
x=inv(A)*B;
disp(x);

OUTPUT:-

>>
3.657
1.349
0.845

Therefore i1=3.657,i2=1.349 and i3=0.845

MATLAB CODE when R2 and R3 are set to 0 is as below

is a betow loop of tor toop 120 x 1レ 2MATLAB CODE:-

A=[13 -13 0;0 -26 6; 0 6 -116];
B=[120;0;0];
x=inv(A)*B;
disp(x);

OUTPUT:-

>>
9.230
0
0

Therefore i1=9.230,i2=0 and i3=0 when R2 and R4 are set to 0

Add a comment
Know the answer?
Add Answer to:
Hi would you mind answering the question? I appreciate your help. (Matlab) - using matlab program...
Your Answer:

Post as a guest

Your Name:

What's your source?

Earn Coins

Coins can be redeemed for fabulous gifts.

Not the answer you're looking for? Ask your own homework help question. Our experts will answer your question WITHIN MINUTES for Free.
Similar Homework Help Questions
  • I need help with part b of #1 and also #2 R_1 = 1.12 Ohm; R_2...

    I need help with part b of #1 and also #2 R_1 = 1.12 Ohm; R_2 = 2.24 Ohm; R_3 = 5.60 Ohm; R_4 = 3.36 Ohm How much current flows through each of the four resistors? I_1 = A I_2 = What is the total current delivered by the battery? What is the voltage across R_1? What are the voltages across each of the three parallel resistors? A I_3 = A I_4 = A

  • Matlab question: Resistive networks are well-represented by linear equations. Consider the DC circuit shown below: Figure...

    Matlab question: Resistive networks are well-represented by linear equations. Consider the DC circuit shown below: Figure 1 This problem can be converted into a system of simultaneous linear equations by applying Kirchhoff's law and Ohm's law. In circuit design, i represents current (measured in amperes, A), V represents voltage (in volts, V), and R represents resistance (in ohms, ohm). Kirchhoff's law states that the sum of the currents entering a node is equal to the sum of the currents leaving...

  • MATLAB MATLAB MATLAB Consider the diagram below V R$ Here, each V represents a change in...

    MATLAB MATLAB MATLAB Consider the diagram below V R$ Here, each V represents a change in voltage (in volts) at a battery, each R represents a resistance in ohms) at a resistor and each I represents a current (in amps) through a wire. These quantities obey two simple laws: 1. Ohm's law: The voltage drop across a resistor is V = IR. 2. Kirchhoff's second law: The sum of all the voltage changes in a closed loop is zero. Using...

  • ri R$ 7:) Here, each V represents a change in voltage (in volts) at a battery,...

    ri R$ 7:) Here, each V represents a change in voltage (in volts) at a battery, each R represents a resistance (in ohms) at a resistor and each I represents a current (in amps) through a wire. These quantities obey two simple laws: 1. Ohm's law: The voltage drop across a resistor is V = IR. 2. Kirchhoff's second law: The sum of all the voltage changes in a closed loop is zero. Using these two laws, we can construct...

  • I only need help with the discussion there are many info that you do not need put I put just in ...

    I only need help with the discussion there are many info that you do not need put I put just in case as well as my data table. please do it as soon as u can Meauements ODeit1 Check2 Meaurements Checi3 Ced Check5 Check6 Measurements Check7 heck8 4058 33 536 1502 1035 979 119478 041 1.0319|0.554972| 64261| 153|15542033681 995781 5266566 1578807 298h 1631 119 1209430016 079096812 0.418135246 0,00032665 011890:004668155728441 0293237 1.291809502 1.22833 1.28 1952 -1116 4281 140616885511984206317 3346 8162 0.78...

ADVERTISEMENT
Free Homework Help App
Download From Google Play
Scan Your Homework
to Get Instant Free Answers
Need Online Homework Help?
Ask a Question
Get Answers For Free
Most questions answered within 3 hours.
ADVERTISEMENT
ADVERTISEMENT
ADVERTISEMENT